Consider the following sample data: x 12 18 20 22 25 y 15 20 25 22 27 Click here for the Excel Data File a. Calculate the covariance between the variables. (Round your intermediate calculations to 4 decimal places and final answer to 2 decimal places.)

Answers

Answer 1

Answer:

The covariance between the variables is 21.10 and the Correlation coefficient is 0.9285.

Step-by-step explanation:

Hence,

Consider The Following Sample Data: X 12 18 20 22 25 Y 15 20 25 22 27 Click Here For The Excel Data File
Consider The Following Sample Data: X 12 18 20 22 25 Y 15 20 25 22 27 Click Here For The Excel Data File

Related Questions

Rectangle ABCD translates 4 units down and 2 units to the right to form rectangle A'B'C'D'. The vertices of rectangle ABCD are labeled in alphabetical order going clockwise around the figure. If AB = 3 units and AD = 5 units, what is the length of B'C'?

Answers

Answer:

The length of BC is 14 units.

Step-by-step explanation:

[tex]hope \: \: it \: \: helps} \beta \alpha \infty [/tex]

The length of B'C' is 0 units.

What is translation?

It is the movement of the shape in left, right, up, and down direction.

The translated shape will have the same shape and shape.

There is a positive value when translated to the right and up.

There is a negative value when translated to the left and down.

We have,

The length of AD = 5 units.

Since the rectangle translates down by 4 units,

The length of A'D' =5 units.

The width of the original rectangle is AB, which is 3 units.

Since the rectangle translates to the right by 2 units,

The width of the new rectangle = 3 units.

Now,

The length of B'C' is the same as the length of AD', which is 5 units.

Subtracting 5 units from 5 units gives us a length of 0 units.

Thus,

The length of B'C' is 0 units.

Learn more about translation here:

https://brainly.com/question/12463306

#SPJ7

Suppose that from a group of 9 men, 1 will be randomly chosen for a dangerous assignment, and suppose that the chosen man will be killed during the assignment with a probability of 1/6. If Mark is one of the 9 men, what is the probability that he will be chosen for the assignment and killed during the assignment

Answers

Answer:

1/54

Step-by-step explanation:

1/9 x 1/6

 Marsha has a bag that contains 4 green marbles, 8 yellow marbles , and 20 red marbles . If she chooses one marble from the bag, what is the probability that the marble is not red?

PLEASE HELP IF YOURE GOOD AT GEOMETRY!!

Answers

Answer:

C. 3/8

HOPE THIS HELPS :)

Answer:

c. 3/8

Step-by-step explanation:

first you need the denomerator by adding all marbles together which equals 32. now for the munerator you need the sum of the green and yellow marbles. this equals 12. so your fraction is 12/32. next we simplify. we can divide both numbers by 4. getting us a fraction of 3/8.

Lance is selling T-shirts for $10 each and hats for $12.50 each. He wants to earn at least $400 per week to cover his expenses. Which graph best represents the number of T-shirts and hats Lance should sell to meet his goal?

Answers

i think you forgot to show the graph

Answer:

Step-by-step explanation:

Find the area of the following shape:

Answers

Answer:

36cm^2

Step-by-step explanation:

total area: 6x(4+3)=42

total area excluding the space: 42-(2x3)=36

Answer:

36 cm squared

Step-by-step explanation:

To solve this problem, I first construct a line. (shown in yellow in the first photo)

I then find the area of the top rectangle. (6 cm * 4 cm = 24 cm squared.)

Next, I find the area of the lower rectangle. But...to do that I have to find the length of the line that I constructed. To do this, I do  6cm-2cm=4cm.

Then I can find the area of the lower rectangle. (4cm*3cm=12cm squared.)

add up the area of both of the rectangles and.........12+24=36 cm squared

What is the reciprocal of tanB in the triangle below?

Right triangle A B C is shown. C is the right angle and side A B is the hypotenuse.
tanC
tanA
tan-1C
tan-1A

Answers

Answer:

Tan A

Step-by-step explanation:

Tan B = opposite / Adjacent = AC / BC

Reciprocal of Tan B = 1 ÷ Tan B

1 ÷ Tan B = 1 ÷ AC/BC = 1 * BC / AC = BC / AC

Reciprocal of Tan B = BC / AC

the reciprocal of tan B is equivalent to :

Tan A = opposite / Adjacent = BC / AC

Hence, the reciprocal of Tan B is Tan A

Answer:

Note: Images are not in order. Check page number on pictures to make sure you have the right Answer.

Have a Good Day! God bless!

Step-by-step explanation:

Question 6 “A”

Question 7 “D”

Question 8 “B”

Question 9 “B”

Question 10 “A”

Note: Answers From 1 to 5 in order here:

Question 1 “ B”

Question 2 “A”

Question 3 “B”

Question 4 “D”

Question 5 “D”

Given points (-3;-6), G(3; -2) and H(6; 1); determine:
(a) The equation of line FG

Answers

Answer:

The equation of line FG is [tex]y = \frac{2}{3}x - 4[/tex]

Step-by-step explanation:

Equation of a line:

The equation of a line has the following format:

[tex]y = mx + b[/tex]

In which m is the slope and b is the y-intercept.

F(-3;-6), G(3; -2)

When we have two points, the slope is given by the change in y divided by the change in x. So

Change in y : -2 - (-6) = -2 + 6 = 4

Change in x: 3 - (-3) = 3 + 3 = 6

Slope: [tex]m = \frac{4}{6} = \frac{2}{3}[/tex]

So

[tex]y = \frac{2}{3}x + b[/tex]

Finding b:

(3; -2) means that when [tex]x = 3, y = -2[/tex]. We use this to find b.

[tex]y = \frac{2}{3}x + b[/tex]

[tex]-2 = \frac{2}{3}(3) + b[/tex]

[tex]2 + b = -2[/tex]

[tex]b = -4[/tex]

The equation of line FG is [tex]y = \frac{2}{3}x - 4[/tex]

0.7(1.5 + y) = 3.5y - 1.47

Answers

Answer:

y = 0.9

Step-by-step explanation:

1.05 + 0.7y = 3.5y - 1.47

-3.5y + 0.7y = -1.47 - 1.05

-2.8y = -2.52

y = 9/10 = 0.9

Answer:

[tex]\textbf{HELLO!!}[/tex]

[tex]0.7\left(1.5+y\right)=3.5y-1.47[/tex]

[tex]1.05+0.7y=3.5y-1.47 \gets \textsl{Expand}[/tex]

[tex]1.05+0.7y-1.05=3.5y-1.47-1.05 \gets Subtract\; 1.05 \from\:both\:sides[/tex]

[tex]0.7y=3.5y-2.52[/tex]

[tex]0.7y-3.5y=3.5y-2.52-3.5y[/tex]

[tex]\mathrm{Subtract\:}3.5y\mathrm{\:from\:both\:sides} \nwarrow[/tex]

[tex]-2.8y=-2.52[/tex]

[tex]\frac{-2.8y}{-2.8}=\frac{-2.52}{-2.8} \hookleftarrow \mathrm{Divide\:both\:sides\:by\:}-2.8[/tex]

[tex]\boxed{\boxed{\underline{\textsf{\textbf{y=0.9}}}}}[/tex]

[tex]\bullet \bullet \bullet \bullet \bullet \bullet \bullet \bullet \bullet \bullet \bullet \bullet \bullet[/tex]

[tex]\textbf{HOPE IT HELPS}[/tex]

[tex]\textbf{HAVE A GREAT DAY!!}[/tex]

Suppose that g(x)= f(x)+ 6. Which statement best compares the graph of g(x) with the graph of f(x)?

A. The graph of g(x) is the graph of f(x) shifted 6 units down.

B. The graph of g(x) is the graph of f(x) shifted to the right.

C. The graph of g(x) is the graph of f(x) shifted 6 units to the left.

D. The graph of g(x) is the graph of f(x) shifted 6 units up.

Answers

Answer:

D

Step-by-step explanation:

The + 6 moves it up 6 units.

The correct answer is (D) "The graph of g(x) is the graph of f(x) shifted 6 units up."

What is the function?

A relationship between a group of inputs and one output is referred to as a function. In plain English, a function is an association between inputs in which each input is connected to precisely one output. A domain, codomain, or range exists for every function. Typically, f(x), where x is the input, is used to represent a function.

When we add a constant to a function, such as in the case of g(x) = f(x) + 6, it will shift the graph of f(x) upward by 6 units.

This is because, for any value of x, the value of f(x) will be added to 6, resulting in a vertical shift of the entire graph.

Option (A) is incorrect because adding 6 to f(x) would shift the graph up, not down.

Option (B) is incorrect because adding a constant to a function does not cause it to shift horizontally.

Option (C) is incorrect because adding 6 to f(x) would shift the graph right, not left.

D. The graph of g(x) is the graph of f(x) shifted 6 units up. Adding a constant term to a function will shift the graph of the function vertically. In this case, adding 6 to f(x) will shift the graph of f(x) upward by 6 units, resulting in the graph of g(x).

Learn more about function here:

https://brainly.com/question/29633660

#SPJ7

Help me with this answer I don’t it

Answers

Answer:

f(-2) = g(-2) this is the answer

A group of 3 boys is sharing 4 small pans of rice krispie treats. What is the total amount of rice krispie treats each boy will get?

Answers

Answer:

each boy will get 1.33333333333 of a pan

Step-by-step explanation:

Answer:

Each boy in that group would get about 1 and 1 third of a pan of treats or 1.33 repeating

Step-by-step explanation:

To find this you would take the numbers of pans (4) and divide it by the number of boys (3).

4 / 3 = 1.33333

Find the missing length indicated

Answers

x = 65

Step-by-step explanation:

cos theta = 25/x

cos theta = x/169

25/x = x/169

x² = 169 x 25

x = 65

The missing length x = 65, using the Pythagoras Theorem.

What is the Pythagoras Theorem?

According to the Pythagoras Theorem, in a right triangle, the square of the hypotenuse is equal to the sum of the squares of the legs.

How to solve the question?

In the question, we are asked to find the value of x.

In the right triangle ABC, by Pythagoras' Theorem,

AC² + BC² = AB²,

or, x² + BC² = (144 + 25)²,

or, BC² = 169² - x² ... (i).

In the right triangle ACD, by Pythagoras Theorem,

AD² + DC² = AC²,

or, 25² + DC² = x²,

or, DC² = x² - 25² ... (ii).

In the right triangle BCD, by Pythagoras Theorem,

BD² + DC² = BC²,

or, 144² + x² - 25² = 169² - x² {Substituting BC² = 169² - x² from (i) and DC² = x² - 25² from (ii)},

or, x² + x² = 169² + 25² - 144² {Rearranging},

or, 2x² = 28561 + 625 - 20736,

or, 2x² = 8450,

or, x² = 4225,

or, x = √4225 = 65.

Thus, the missing length x = 65, using the Pythagoras Theorem.

Learn more about the Pythagoras Theorem at

https://brainly.com/question/231802

#SPJ2

Which diagram represents the hypothesis of the converse of corresponding angles theorem?

Answers

Answer:

the first diagram

Step-by-step explanation:

first one

HELP HELP HELP
Solve this

Answers

Answer:

What is the cos theta for, i would use sin to solve for theta and then we would get 41.25 degrees.

Step-by-step explanation:

HELP ME PLEASE IF YOU DO YOU WILL GET BRAINLESS AND PLEASE EXPLAIN THE BEST YOU CAN

Answers

Answer:

<3=75°

Step-by-step explanation:

Angle 3 and angle 2x+95 are supplementary( supplementary angles add up to 180°)

So <3+2x+95=180

<3+2x=180-95

<3+2x=85( let's call this equation 1)

Next, angle 5 and angle 8x+71 are opposite angles (opposite angles are equal) therefore <5=8x+71

Now, <3 and <5 are co-interior angles(co-interior angles are supplementary)

So <3+8x+71=180

<3+8x=180-71=109

Thus, <3+8x=109(let's call this equation 2)

Now solving equation 1 and 2 simultaneously:

Make <3 the subject of equation 1

<3=85-2x

Put <3=85-2x into equation 2

85-2x+8x=109

6x=24

x=24/6=4

Now, remember that angle 2x+95 becomes

2(4)+95

8+95=103°

Therefore<3=180-105=75°

how would I classify a triangle which has a angle of 49 and 82, acute, right, or obtuse?

Answers

9514 1404 393

Answer:

  acute

Step-by-step explanation:

The third angle is ...

  180° -49° -82° = 49°

So, the triangle has two angles the same, 49°. When two angle are the same, the triangle is an isosceles triangle.

The largest angle, 82°, is less than 90°, so is an acute angle. The classification acute, right, or obtuse is based on the measure of the largest angle.

The triangle is an acute isosceles triangle.

to train for a race, you plan to run 1 mile the first week and double the number of miles each week for five weeks. How many miles will you run for the 5th week. math problem

Answers

Answer:

16 Miles

Step-by-step explanation:

For every week you simply multiply the number of miles from the previous week by 2, therefore

Week 1: 1

Week 2: 2

Week 3: 4

Week 4: 8

Week 5: 16


Ibrahim heeft een bijbaantje op de markt. Hij berekent zijn inkomsten met de formule
inkomsten in €=5+3,50 x tijd in uren. Leg de formule uit.

Answers

Answer:

Ibrahim gets 5 fixed and 3.5 per hour.

Step-by-step explanation:

Ibrahim has a side job at the market. He calculates his income with the formula income in € = 5 + 3.50 x,  time in hours. Explain the formula.

Here, the fixed income is 5.

the income per hour is 3.5.

So, Ibrahim gets 5 fixed and 3.5 per hour.

11x+7y=17
solve for y

Answers

[tex]\implies {\blue {\boxed {\boxed {\purple {\sf {\: y = \frac{17 - 11x}{7} }}}}}}[/tex]

[tex]\large\mathfrak{{\pmb{\underline{\red{Step-by-step\:explanation}}{\red{:}}}}}[/tex]

[tex]\\11x + 7y = 17[/tex]

[tex] \\➺ \: 7y = 17 - 11x[/tex]

[tex]\\➺ \: y = \frac{17 - 11x}{7} [/tex]

[tex]\bold{ \green{ \star{ \orange{Mystique35}}}}⋆[/tex]

Simplify:......................................................

Answers

Answer:

...

Step-by-step explanation:...

The cut off number is 2x-1

Determine whether the following event is mutually exclusive or not mutually exclusive.

Choosing a student who is a mathematics major or a business major from a nearby university to participate in a research study. (Assume that each student only has one major.)

Answers

Answer:

The event is mutually exclusive.

Step-by-step explanation:

Mutually exclusive events are events that cannot exist simultaneously.

Thus, events that are not mutually exclusive can exist simultaneously.

Since each student only has one major, a single student cannot be both a mathematics major and a business major.

So, the event is mutually exclusive.

Joe drives for 3 hours and covers 201 miles. In miles per hour, how fast was he driving?​

Answers

Answer:

67 mph

Step-by-step explanation:

201/3 = 67

Six consecutive numbers add up to a total of 69.what is the highest of these numbers?

Answers

Answer:

14

Step-by-step explanation:

9+10+11+12+13+14=69

The [tex]HIGHEST[/tex] of these numbers is [tex]14[/tex]

Let the [tex]CONSECUTIVE[/tex] numbers be : [tex]a, a+1, a +2,a+3,a+4,a+5[/tex]

Taking the sum :

[tex]a + a + 1 + a + 2 + a + 3 + a + 4 + a + 5 = 69\\6a + 15 = 69\\6a = 69 - 15\\6a = 54\\a = 54 / 6\\a = 9[/tex]

[tex]HIGHEST[/tex] value = [tex]a + 5 = 9 + 5 = 14[/tex]

Hence, [tex]HIGHEST[/tex] value = [tex]14[/tex]

Learn more : https://brainly.com/question/15974141

Let r be the binomial random variable corresponding to the number of people that will live beyond their 90th birthday,
r ≥ 15.


We want to find
P(r ≥ 15)
using the normal approximation given 625 trials and a probability of a 4.4% success on a single trial.

Answers

Answer:

P(r ≥ 15) = 0.9943.

Step-by-step explanation:

We use the normal approximation to the binomial to solve this question.

Binomial probability distribution

Probability of exactly x successes on n repeated trials, with p probability.

Can be approximated to a normal distribution, using the expected value and the standard deviation.

The expected value of the binomial distribution is:

[tex]E(X) = np[/tex]

The standard deviation of the binomial distribution is:

[tex]\sqrt{V(X)} = \sqrt{np(1-p)}[/tex]

Normal probability distribution

Problems of normally distributed distributions can be solved using the z-score formula.

In a set with mean [tex]\mu[/tex] and standard deviation [tex]\sigma[/tex], the z-score of a measure X is given by:

[tex]Z = \frac{X - \mu}{\sigma}[/tex]

The Z-score measures how many standard deviations the measure is from the mean. After finding the Z-score, we look at the z-score table and find the p-value associated with this z-score. This p-value is the probability that the value of the measure is smaller than X, that is, the percentile of X. Subtracting 1 by the p-value, we get the probability that the value of the measure is greater than X.

When we are approximating a binomial distribution to a normal one, we have that [tex]\mu = E(X)[/tex], [tex]\sigma = \sqrt{V(X)}[/tex].

625 trials and a probability of a 4.4% success on a single trial.

This means that [tex]n = 625, p = 0.044[/tex]

Mean and standard deviation:

[tex]mu = E(X) = np = 625*0.044 = 27.5[/tex]

[tex]\sigma = \sqrt{V(X)} = \sqrt{np(1-p)} = \sqrt{625*0.044*0.956} = 5.13[/tex]

P(r ≥ 15)

Using continuity correction, this is [tex]P(r \geq 15 - 0.5) = P(r \geq 14.5)[/tex], which is 1 subtracted by the p-value of Z when X = 14.5. So

[tex]Z = \frac{X - \mu}{\sigma}[/tex]

[tex]Z = \frac{14.5 - 27.5}{5.13}[/tex]

[tex]Z = -2.53[/tex]

[tex]Z = -2.53[/tex] has a p-value of 0.0057

1 - 0.0057 = 0.9943

So

P(r ≥ 15) = 0.9943.

Which number's estimate written as a single digit times a power of 10 will have a negative exponent?

Answers

105 i hope this helps if not then i’m sorry

Kin travels 440 miles by train at an average speed of 110 mph.
Ayaan flies the same distance at an average speed of 880 mph.
Find the difference between their travel times.
Give your answer in hours.

Answers

Answer:

3 1/2 hours

Step-by-step explanation:

time = distance/speed

440/110 = 4 hours

440/880 = 1/2 hours

4 minus 1/2 is 3 1/2

Answer:

hi

Step-by-step explanation:

i just need some points sorry not sorry

21(2-y)+12y=44 find y​

Answers

Answer: y= -2/9
Explanation:
21(2-y)+12y=44
42-21y+12y=44
42-9y=44
-9y=2
y=-2/9

Answer:

[tex]\textbf{HELLO!!}[/tex]

[tex]21\left(2-y\right)+12y=44[/tex]

[tex]42-21y+12y=44[/tex]

[tex]~add ~similar\:elements[/tex]

[tex]42-9y=44[/tex]

[tex]Subtract~42~from~both~sides[/tex]

[tex]42-9y-42=44-42[/tex]

[tex]-9y=2[/tex]

[tex]Divide\:both\:sides\:by\:}-9[/tex]

[tex]\frac{-9y}{-9}=\frac{2}{-9}[/tex]

[tex]y=-\frac{2}{9}[/tex]

----------------------

hope it helps...

have a great day!

Need help finding the factor of 2y^2-2y-4

Answers

Answer:

hope it helps you............

Answer:

2(y - 2)(y + 1)

Step-by-step explanation:

Given

2y² - 2y - 4 ← factor out 2 from each term

= 2(y² - y - 2) ← factor the quadratic

Consider the factors of the constant term (- 2) which sum to give the coefficient of the y- term (- 1)

The factors are - 2 and + 1, since

- 2 × 1 = - 2 and - 2 + 1 = - 1 , then

y² - y - 2 = (y - 2)(y + 1)

Then

2y² - 2y - 4 = 2(y - 2)(y + 1) ← in factored form

Moving to another question will save this response.
1 points
Save Answer
Question 12
Mr Espent 65% of his salary on household expenses, and 15% of the remainder on travelling expenses and was finally left with R9 500. How much was his salary?​

Answers

Answer:

rs.1680.67

Step-by-step explanation:

His salary = x

remaining % = 100 - 65 = 35%

= 100 - 15 = 85%

x × 35/100 × 85/100 = 500

x = 1680.67

A road has a scale of 1:50 000 The length of a road on the map is 8.5cm.Work out the length of the real road in kilometres

Answers

Answer:

ok so

8.5*150000

1275000 cm into kilometers is

12.75 kilometers

Hope This Helps!!!

Other Questions
Say that investment increases by $60 for each interest rate drop of 1 percent. Say also that the expenditures multiplier is 4. If the money multiplier is 5, and each 5-unit change in the money supply changes the interest rate by 1 percent, what open market policy would you recommend to increase income by $240 ayudddaaa es para en den tro de 3 munutos doy corona 2 extra large chocolate shakes contain 85 ounces of ice cream. How many ounces of ice cream are there in 12 extra large chocolate shakes?plz show work How many dimensions does a plane have?OA. ThreeB. ZeroO C. TwoD. One Hello, I need help with this math question please What is trench warfare, and why was so much of World War I dominated by this method of fighting? Which instrument changes kinetic energy into electrical energy?electric motorhair drierelectric belldynamo How can business activity affect the government? What is the answer of How does an indicator show the difference between an acid and an alkali? Where does the exchange of oxygen (from the air) with carbon dioxide (from the cells) take place? A. Where are the bronchi meet the bronchiolesB. Where the blood meets the cellsC. Where the nasal passages meet the airD. Where are the alveoli meet capillaries Jiminys Cricket Farm issued a bond with 25 years to maturity and a semiannual coupon rate of 4 percent 3 years ago. The bond currently sells for 108 percent of its face value. The companys tax rate is 22 percent. When thinking about job, career, and calling, your results from your profile, and choosing a college major , what comes to mind?PLEASE HELP ME THIS IS DIE TODAY AND I REALLY NEED HELP Write a letter to someone who had bullied you in the past, including detail. Brainliest! Which of the following is true regarding the sequence graphed below?an353025 (5, 25)2015- (4. 16)10 - (3,9)5 (2, 4)1 2 345 67 89nThe sequence is arithmetic because the terms have a common difference.O The sequence is arithmetic because the terms do not have a common difference THE IMPORTANCE OF INFORMATION IN MARKETING When starting an exercise program, shrinking your goals down to realistic and attainable goals like just moving for 10 minutes a day, is better than setting a lofty goal of losing 10 pounds a week. TRUE OR FALSE Do private practice doctors and hospital doctors have the same distribution of working hours? Suppose that a sample of 100 private practice doctors and 150 hospital doctors are selected at random and asked about the number of hours a week they work. You may use technology to compute your answers. 2030304040505060Total Private Practice16403894 Hospital84459111 Total248497205 a cylindrical barrel (drum) contains 42 gallons of oil. the diameter of this barrel is 18 inchesdetermine the radius of a barrel (drum) in centimetre. What environmental factor was important for theCambrian "explosion of life?